Đến nội dung

Kamii0909

Kamii0909

Đăng ký: 26-08-2016
Offline Đăng nhập: 26-07-2022 - 16:44
**---

#689342 Đề Thi Trại Hè Hùng Vương 2017

Gửi bởi Kamii0909 trong 03-08-2017 - 00:23

Bài 4.
$a+b^2 \mid a^2+b$ nên $a \geq b$ 

$a=b$ thì $2 \mid a(a+1)$, do đó $2^x=a(a+1)$
Nếu $a\geq 2$ thì $a(a+1)$ sẽ có ước khác 2(vô lý)
Vậy $a=b=1$
Xét $a > b \geq 1$
$a+b^2 \mid a^2+b$
$\Leftrightarrow a+b^2 \mid a(a+b^2)-b(ab-1)$
Mà $(a,ab-1)=1$ nên $a+b^2 \mid b(a+b^2)-b^3-1$
Hay $p^x=a+b^2 \mid (b+1)(b^2-b+1)$
Dễ thấy $a+b^2 >b+1$ và $a+b^2>b^2-b+1$ nên $p|(b+1,b^2-b+1)$
Từ đó có $p=3$
Mà $9 \nmid b^2-b+1$ nên $3^{x-1} \mid b+1$
$\Rightarrow b \geq 3^{x-1}-1 = \dfrac{a+b^2}{3} -1$
Hiển nhiên điều này sai với $b \geq 3$, mà $3^{x-1} \mid b+1$ nên $b=2, a=5$




#675570 $$\prod \left( \dfrac{a}{b}+2...

Gửi bởi Kamii0909 trong 28-03-2017 - 22:52

Cho các số thực dương $a,b,c$. Cmr
$$ ( \dfrac{a}{b}+2)( \dfrac{b}{c} +2)( \dfrac{c}{a}+2 ) + \dfrac{117(ab+bc+ca)}{4(a^2+b^2+c^2)} \geq \frac{107}{2}$$


#674391 $100+9abc \geq 17(ab+bc+ca)$

Gửi bởi Kamii0909 trong 15-03-2017 - 23:10

Cho $a,b,c \in [1,2]$ thoả $a+b+c=5$. 

Chứng minh rằng 

$$100+9abc \geq 17(ab+bc+ca)$$




#674386 CMR: $\sqrt{a + b} + \sqrt{b + c} + \...

Gửi bởi Kamii0909 trong 15-03-2017 - 22:44

Cho a, b, c là các số dương thay đổi và a + b + c = 4. CMR : $\sqrt{a + b} + \sqrt{b + c} + \sqrt{c + a} > 4$

*P/s: Xin lỗi các bạn, mình đã sửa đề :)

KMTTQ, $a \geq b \geq c$

Đpcm $$\Leftrightarrow \sum \sqrt{a+b} \geq 2 \sqrt{a+b+c}$$

$$\Leftrightarrow \sqrt{b+c} \geq \frac{c}{\sqrt{a+b+c}+\sqrt{a+b}} +\frac{b}{\sqrt{a+b+c}+\sqrt{a+c}}$$

Có $$\sqrt{a+b+c}+\sqrt{a+b} \geq \sqrt{a+b+c}+\sqrt{a+c} \geq \sqrt{b+c}$$

Vậy $$R.H.S \leq \frac{b+c}{\sqrt{b+c}}=L.H.S$$

Ta có điều phải chứng minh.

Spoiler




#674165 CMR:a, $\sum \frac{1}{a^{2}-a+1}...

Gửi bởi Kamii0909 trong 13-03-2017 - 19:36

vì sao ạ?

Ý bạn là sao? Cứ khai triển ra là thấy nó tương đương thôi mà?


#673908 CMR:a, $\sum \frac{1}{a^{2}-a+1}...

Gửi bởi Kamii0909 trong 10-03-2017 - 21:47

1. abc=1, a,b,c dương
CMR:a, $\sum \frac{1}{a^{2}-a+1} \leq 3$
b, $\sum \frac{12a+7}{2a^{2}+1}\leq 19$

Bài 1 khó cả 2 câu.
a. Bđt cần cm tương đương với
$$\sum \frac{(2a-1)^2}{a^2-a+1} \geq 3$$
Áp dụng bđt C-S ta phải cmr
$$\frac{(2a+2b+2c-3)^2}{a^2+b^2+c^2-a-b-c+3} \geq 3$$
Đặt $p=a+b+c=x^2,q=ab+bc+ca$
Ta phải chỉ ra $p^2-9p+6q \geq 0$
Chú ý bđt thông dụng $q^2 \geq 3pr=3p=3x^2$
Bài toán đưa về $x(x- \sqrt{3})^2(x+2 \sqrt{3}) \geq 0$
Hiển nhiên đúng.
Spoiler


b. Viết lại bđt
$$ \sum \frac{(3a-1)^2}{2a^2+1} \geq 4$$
Áp dụng C-S và ta đi cmr
$$\frac{9(a+b+c-1)^2}{2(a^2+b^2+c^2)+3} \geq 4$$
$$\Leftrightarrow a^2+b^2+c^2+18(ab+bc+ca) -18(a+b+c)-3 \geq 0$$
$$\Leftrightarrow a^2+b^2+18(c-1)(a+b) +c^2-18c+18ab-3 \geq 0$$
KMTTQ, $c \geq 1$, Đặt $c=x^2$
Khi đó $$ L.H.S \geq 20ab+36(c-1) \sqrt{ab} +c^2-18c-3 \geq 0$$
$$\Leftrightarrow \frac{20}{x^2} +\frac{36(x^2-1)}{x} +x^4-18x^2-3 \geq 0$$
$$\Leftrightarrow (x-1)^2(x-2)^2(x+1)(x+5) \geq 0$$
Hiển nhiên đúng.
Spoiler



#673764 $a+b+c+\sqrt{a}+\sqrt{b}+\sqrt{c...

Gửi bởi Kamii0909 trong 08-03-2017 - 22:16

Ta phát biểu và chứng minh bổ đề sau.
Nếu $a,b,c>0$ và $a^4b^4+b^4c^4+c^4a^4=3$ thì $a^3+b^3+c^3 \geq 3$

Từ đánh giá $a^3+b^3+1 \geq 3ab$ và giả thiết ta có $$\sum xy(x+y+1) \geq 9$$ trong đó $x=a^3,y=b^3,c=z^3$
Giả sử rằng $p=x+y+z \leq 3$
Trước hết,ta sẽ cmr $r \geq \frac{4q-9}{3}$
Theo Schur, $r \geq \frac{p(4q-p^2)}{9}$
Kết quả nếu trên sẽ được chứng minh nếu ta chỉ ra rằng $p(4q-p^2) \geq 12q-27 \Leftrightarrow (3-p)(p^2+3p+9-4q) \geq 0$.
Bất đẳng thức cuối đúng do $9 \geq p^2 \geq 3q$.
Vậy $3r \geq 4q-9$
Mà theo trên ta có $4q-9 \geq pq+q-9 \geq 3r$ nên dấu bằng phải xảy ra hay $x=y=z=1,a=b=c=1$. Khi đó $a^3+b^3+c^3=3 \geq 3$
Bổ đề được chứng minh.
Quay lại bài toán.
Ta có $a+ \sqrt{a} \geq 2 \sqrt[4]{a^3}$
Bài toán trở thành bổ đề cho $( \sqrt[4]{a},\sqrt[4]{b},\sqrt[4]{c})$


#673603 VMF's Marathon Bất Đẳng Thức Olympic

Gửi bởi Kamii0909 trong 06-03-2017 - 22:30

cho a, b, c >0 chứng minh
$\frac{a}{a+b}+\frac{b}{b+c}+\frac{c}{c+a}\geq 1+\sqrt{\frac{2abc}{(a+b)(b+c)(c+a)}}$

Do tính thuần nhất ta có thể cho $c=1$. Khi đó bất đẳng thức cần chứng minh tương đương
$$a^2b+b^2+a+ab\geq \sqrt{2ab(a+b)(a+1)(b+1)}$$
Bình phương lên và biến đổi nó tương đương với $a^4b^2+b^4+a^2 \geq 3a^2b^2$
Đúng theo AM-GM


#673316 CMR: $\sum \frac{a}{\sqrt{b+c}...

Gửi bởi Kamii0909 trong 03-03-2017 - 01:58

Có $$\sum \frac{a}{\sqrt{\frac{3}{2}(b+c)(a+b+c)}} \geq \sum \frac{4a}{2a+5b+5c} \geq \frac{4(a+b+c)^2}{2(a+b+c)^2+6(ab+bc+ca)} \geq 1$$




#671421 $\sum (a^3-b^3)^2 \geq 3abc(a-b)(b-c)(c-a)$

Gửi bởi Kamii0909 trong 13-02-2017 - 02:50

Cho $a,b,c$ là các số thực thỏa mãn $a^5b+b^5c+c^5a=a^4b^2+b^4c^2+c^4a^2$
Chứng minh rằng
$(a^3-b^3)^2+(b^3-c^3)^2+(c^3-a^3)^2 \geq 6abc(a-b)(b-c)(c-a)$


#670624 $\sum \frac{1}{\sqrt{a+2b+6}}\leq 1$

Gửi bởi Kamii0909 trong 07-02-2017 - 13:52

Cho các số thực dương $a, b, c$ thỏa mãn $abc=1.$ Chứng minh rằng $\frac{1}{\sqrt{a+2b+6}}+\frac{1}{\sqrt{b+2c+6}}+\frac{1}{\sqrt{c+2a+6}}\leq 1.$


$$\sum_{cyc} \frac{1}{\sqrt{a+2b+6}} \leq \sqrt{ 3 \sum_{cyc} \frac{1}{a+2b+6}} \leq \sqrt{ 3 \left( \dfrac{4}{9} \sum_{cyc} \dfrac{1}{a+2b+3} + \frac{1}{9} \right)} \leq \sqrt{3 \left( \dfrac{4}{9} \sum_{cyc} \dfrac{1}{2( \sqrt{ab}+ \sqrt{b}+1)} + \frac{1}{9} \right)} = 1$$


#670566 $ \frac{a^2b+b^2c+c^2a}{a^3+b^3+c^3} \leq \frac{ab+bc+ca}...

Gửi bởi Kamii0909 trong 06-02-2017 - 20:14

Cho các số thực không âm $a,b,c$. Chứng minh rằng
$$ \frac{a^2b+b^2c+c^2a}{a^3+b^3+c^3} \leq \frac{ab+bc+ca}{a^2+b^2+c^2} + \frac{3 \sum (a-b)^2}{4 (a+b+c)^2}$$


#670560 Chứng minh rằng: $\left | \frac{a^{3}-b^{...

Gửi bởi Kamii0909 trong 06-02-2017 - 19:24

Cho a, b, c là các số thực dương. Chứng minh rằng:
$\left | \frac{a^{3}-b^{3}}{a+b}+\frac{b^{3}-c^{3}}{b+c}+\frac{c^{3}-a^{3}}{c+a} \right |\leqslant \frac{1}{4}\left [(a-b)^{2}+(b-c)^{2}+(c-a)^{2} \right ]$

Hằng số tốt nhất cho bất đẳng thức này khá xấu và có thể tìm bằng dồn biến toàn miền.
Cho $$a=0,b=2,c=1+ \sqrt{3}+\sqrt{2} \cdot 3^\frac{1}{4}$$ thì $k \geq \sqrt{\dfrac{2 \cdot \sqrt{3}-9}{9}}$


#670339 Chứng minh rằng: $\left | \frac{a^{3}-b^{...

Gửi bởi Kamii0909 trong 29-01-2017 - 14:07

Có vẻ như $\frac{1}{4}$ chưa phải hằng số tốt nhất.
Bình phương lên, điều phải chứng minh tương đương.
$$4\prod(a-b)^2 (ab+bc+ca)^2 \leq \prod (a+b)^2 (\sum a^2-bc)^2$$
Đổi về pqr.
$$ \dfrac{4q^2}{27} [4(p^2-3q)^3 -(2p^3-9pq+27r)^2] \leq (pq-r)^2(p^2-3q)^2 $$
$$L.H.S \leq \dfrac{16q^2(p^2-3q)^3}{27}$$
Ta quy điều phải chứng minh về
$$\dfrac{16q^2(p^2-3q)}{27} \leq (pq-r)^2$$
Có $$pq-r \geq \dfrac{8pq}{9}$$
Thay vào và biến đổi, bất đẳng thức tương đương với
$$q^2(\frac{p^2}{3} +3q) \geq 0$$
Hiển nhiên đúng.


#669788 $ \sum \sqrt{a+b+\sqrt{ca}+\sqrt{cb}} \geq k(...

Gửi bởi Kamii0909 trong 24-01-2017 - 22:29

Cho các số thực không âm $a,b,c$ thỏa mãn $a^2+b^2+c^2=2(ab+bc+ca)$
Tìm hằng số k tốt nhất sau cho bất đẳng thức sau luôn đúng $$ \sum \sqrt{a+b+\sqrt{ca}+\sqrt{cb}} \geq k(\sum \sqrt{a})$$